Módulo 1 Cirugía Flashcards

1
Q

A 22-year-old man is stabbed in the right chest with a 5-cm-long knife blade. On arrival at the emergency department, he is wide awake and alert. He is speaking with a normal tone of voice but complaining of shortness of breath. The right hemithorax is hyperresonant to percussion and has no breath sounds; the rest of the initial survey is negative. His blood pressure is 110/75 mm Hg, pulse is 86/min, and venous pressure is 3 cm H2O. Pulse oximetry shows a saturation of 85%. Which of the following is the most appropriate next step in patient care?

(A) Infusion of 2 L Ringer’s lactate
(B) Securing an airway by orotracheal intubation
(C) Immediate insertion of a needle into the right pleural space
(D) Chest x-ray and insertion of a chest tube
(E) Sonographically guided evacuation of the pericardial sac

A

Respuesta: D

The correct answer is D. A penetrating wound to the chest will produce either a pneumothorax, a hemothorax, or both. The absence of breath sounds confirms that one of those has occurred, and the hyperresonance to percussion indicates that air is present. The patient’s good vital signs indicate that there is time to do the proper diagnostic study (chest x-ray). The appropriate treatment for a pneumothorax is placement of a chest tube.

Infusion of 2 L Ringer lactate (choice A) would have been appropriate if the findings had suggested hemothorax (as evidenced by dullness to percussion), and he had been bleeding (as evidenced by low blood pressure and a fast pulse). A patient who is fully awake and alert, and who is speaking in a normal tone of voice, has an airway and can maintain it (compare with choice B).

Immediate insertion of a needle into the right pleural space (choice C) would be appropriate management for a tension pneumothorax. If the patient had a tension pneumothorax, he would have been in shock and severe respiratory distress, and the mediastinum would have been shifted (evidenced by tracheal deviation).

Sonographically guided evacuation of the pericardial sac (choice E) would be appropriate management for pericardial tamponade, which is not present in this patient. If the patient had developed tamponade, he would have been in shock, with a high central venous pressure (or distended veins).

How well did you know this?
1
Not at all
2
3
4
5
Perfectly
2
Q

A 35-year-old man comes to the physician because of persistent dull perineal pain and dysuria for 6 months. The patient denies urinary tract infections or urethral discharge. His temperature is 37.0 C (98.6 F). On digital rectal examination, the prostate is slightly tender and boggy but not enlarged or indurated. Urinalysis is normal. Expressed prostatic secretions show the following:

Leukocytes 30 cells/high power field
Bacteria None

Cultures of prostatic secretion and urine are negative for bacteria. Which of the following is the most likely diagnosis?

(A) Acute cystitis
(B) Acute prostatitis
(C) Chronic bacterial prostatitis
(D) Chronic nonbacterial prostatitis
(E) Prostatodynia

A

Respuesta: D

The correct answer is D. Chronic nonbacterial prostatitis is characterized by persistent irritative voiding symptoms, such as dysuria and perineal discomfort, and leukocytes (especially foamy macrophages) in expressed prostatic secretion. No bacteria, however, are isolated from cultures of urine or prostatic secretions. This condition is believed to be of a noninfectious nature and possibly autoimmune-mediated. Treatment is based on symptomatic relief with sitz baths and anti-inflammatory agents. However, some authors recommend a trial with erythromycin.

Acute cystitis (choice A) is usually infectious, so that irritative voiding symptoms are associated with positive urine cultures. Coliform bacteria are the usual pathogens. In men, prostatic hyperplasia is the most common predisposing factor.

Acute prostatitis (choice B) is due to bacterial infection. Perineal pain, irritative voiding symptoms, extreme tenderness on digital rectal examination, and fever are the presenting symptoms. Urine cultures are positive for the offending agents, which are gram- negative rods (Escherichia coli and Pseudomonas aeruginosa).

Chronic nonbacterial prostatitis must be differentiated from chronic bacterial prostatitis (choice C). Both disorders present with similar symptomatology, but chronic bacterial prostatitis is associated with positive bacterial cultures of expressed prostatic secretions. Gram- negative rods are the most common pathogens. Treatment is based on antibiotic therapy as determined by susceptibility tests on the isolated organisms.

Prostatodynia (choice E) is an obscure entity characterized by dull perineal discomfort and pain mimicking chronic prostatitis. Microscopic examination and cultures of prostatic secretions, however, are negative for leukocytes and bacteria. The designation itself is a misnomer, since the prostate is entirely normal. The disease seems to be related to dysfunctional contractility of the bladder detrusor muscle, the sphincter, and/or the urethra. The treatment is symptomatic and includes alpha-blocking agents, diazepam (as a myorelaxant), biofeedback techniques, and sitz baths.

How well did you know this?
1
Not at all
2
3
4
5
Perfectly
3
Q

A man involved in a high-speed, head-on automobile collision arrives at the emergency department in a deep coma. His pupils react poorly to light but are of equal size. An airway is placed, and the patient is sent for CT scan of the head with extension to the neck. The study shows no cervical spine fractures, but does reveal a small, crescent-shaped hematoma on the right side, with no deviation of the midline structures. Which of the following is the most appropriate next step in management?

(A) High-dose steroids
(B) Hyperventilation, diuretics, and fluid restriction
(C) Systemic vasodilators and alpha blockers
(D) Surgical evacuation of his epidural hematoma
(E) Surgical evacuation of his subdural hematoma

A

Respuesta: B

The correct answer is B. A crescent-shaped hematoma is seen in acute subdural hematoma, whereas acute epidural hematoma produces a biconvex, lensshaped collection. The diagnosis is therefore acute subdural hematoma, but the hematoma is not displacing structures, either clinically (pupils are of equal size) or radiologically. Evacuation is not a priority. The neurologic damage resulted from the initial blow and could be compounded by a subsequent increase in intracranial pressure. Therapy should therefore be directed at preventing such an increase, i.e., hyperventilation, diuretics, and fluid restriction.

Immediate administration of high-dose steroids (choice A) may result in a better long-term outcome in spinal cord injuries. Steroids also lower the elevated intracranial pressure caused by brain tumors; however, for reasons that we do not understand, these agents do not do so in cases of increased intracranial pressure caused by trauma.

Vasodilators would increase intracranial pressure, whereas alpha blockers (choice C) would produce systemic hypotension and further reduce brain perfusion.

The patient does not have an epidural hematoma (choice D) as evidenced by the lack of a biconvex, lens-shaped collection on CT.

Surgical evacuation of his subdural hematoma (choice E) is not indicated if no damage has been caused by the hematoma, since there is no midline shift or anisocoria (inequality of pupils).

How well did you know this?
1
Not at all
2
3
4
5
Perfectly
4
Q

An otherwise healthy 28-year-old man comes to his physician because of painless enlargement of the right testis. He began to feel a sensation of heaviness in the right hemiscrotum approximately 6 months ago. Physical examination reveals diffuse enlargement of the right testis, but it is difficult to determine whether this is due to an intratesticular or extratesticular lesion. Which of the following is the most appropriate next step in diagnosis?

(A) CT scanning
(B) Serum levels of hCG, alpha-fetoprotein, and LDH
(C) Scrotal ultrasonography
(D) Needle biopsy
(E) Inguinal orchiectomy

A

Respuesta: C

The correct answer is C. Ultrasonography is the most sensitive and least expensive method to discriminate between testicular and extratesticular masses. However, a physician should remember to first use a simple transillumination test for such a differential diagnosis. Fluid collections within the vaginal sac transilluminate, whereas testicular masses do not.

CT scanning (choice A) is used to determine the spread of testicular tumors within the abdominal and thoracic cavity, but is of no use in the initial diagnosis of scrotal masses.

Serum levels of hCG, alpha-fetoprotein, and LDH (choice B) are important adjunct parameters in the diagnosis and subsequent management of testicular neoplasms. LDH may be elevated in seminomas and nonseminomas, alpha-fetoprotein is elevated in nonseminomas (especially yolk sac tumors), and hCG is elevated in nonseminomas (especially choriocarcinomas).

Needle biopsy (choice D) is not an adequate diagnostic tool in this case. It may be used in the evaluation of azoospermia related to infertility problems.

Inguinal orchiectomy (choice E) is performed once ultrasonography has established that scrotal enlargement is caused by an intratesticular tumor. This allows the most accurate pathologic diagnosis and appropriate management.

How well did you know this?
1
Not at all
2
3
4
5
Perfectly
5
Q

A 19-year-old gang member is shot in the abdomen with a .38 caliber revolver. The entry wound is in the epigastrium, to the left of the midline. The bullet is lodged in the psoas muscle on the right. He is hemodynamically stable, and the abdomen is moderately tender. Which of the following is the most appropriate next step in diagnosis?

(A) Close clinical observation
(B) Emergency ultrasound
(C) CT scan of the abdomen
(D) Diagnostic peritoneal lavage
(E) Exploratory laparotomy

A

Respuesta: E

The correct answer is E. The abdomen is full of important structures that should not have holes in them: solid organs that can bleed, and hollow viscera that will spill “evil fluids” into the peritoneal cavity. Thus, the rule for abdominal gunshot wounds is simple: an exploratory laparotomy should be done in every case, before there are obvious signs of either bleeding or peritonitis.

Clinical observation alone (choice A) is not wise, since the risk of complications will increase the longer one waits.

Ultrasound (choice B), CT of the abdomen (choice C), and diagnostic peritoneal lavage (choice D) are used to assess the extent of internal damage in blunt abdominal trauma. They would be of little benefit in an abdominal gunshot wound.

How well did you know this?
1
Not at all
2
3
4
5
Perfectly
6
Q

A multiple trauma patient receives 14 units of packed red cells and several liters of Ringer’s lactate solution during a laparotomy for multiple intra-abdominal injuries. The surgeons note that blood is oozing from all dissected raw surfaces, as well as from his IV line sites. His core temperature is normal. Which of the following is the most appropriate next step in management?

(A) Proceed with surgery and give blood transfusions as needed
(B) Obtain a stat coagulation profile to guide specific therapy
(C) Empiric administration of fresh frozen plasma and platelet packs
(D) Abort the operation and close the abdomen with towel clips
(E) Leave the abdomen open and covered with mesh until coagulation parameters can be corrected

A

Respuesta: C

The correct answer is C. In the setting of massive blood loss and multiple transfusions (more than 12 units of packed red cells), the development of coagulopathy is almost predictable. Packed red cells contain virtually no viable platelets and only a very small concentration of clotting factors. Prophylactic administration of clotting factors has not proven to be advantageous, but once the coagulopathy occurs, a shotgun approach to provide fresh frozen plasma and platelet packs is indicated.

Ignoring the coagulopathy and continuing to operate and transfuse (choice A) would be doomed to failure. Surgeons can ligate or cauterize big vessels but cannot do the same for capillaries. Proper clotting is indispensable in all surgical operations.

Although it would be more elegant to determine exactly what is missing, under these circumstances there is no time to do the detailed studies (choice B).

If hypothermia and acidosis had also developed, a more drastic approach would have been necessary: stop the operation and close the abdomen temporarily (choice-D).

Closing with a mesh (choice E) is indicated when an abdominal compartment syndrome occurs-it has nothing to do with coagulopathy.

How well did you know this?
1
Not at all
2
3
4
5
Perfectly
7
Q

A 75-year-old man slips and falls at home, hitting his right chest wall against the kitchen counter. He has an area of exquisite pain to direct palpation over the seventh rib, at the level of the anterior axillary line. A chest x-ray film confirms the presence of a rib fracture, with no other abnormal findings. Which of the following is the most appropriate initial step in management?

(A) Supplemental oxygen to compensate for hypoventilation
(B) Systemic narcotic analgesics
(C) Binding of the chest to limit motion
(D) Intercostal nerve block to minimize pain
(E) Open reduction and internal fixation to accelerate healing

A

Respuesta: D

The correct answer is D. A rib fracture can be a serious injury in the elderly, because the pain prevents full inspiration, atelectasis ensues, and eventually pneumonia develops and may cause significant morbidity and mortality. The key to the treatment is to eliminate the pain without interfering with ventilation. An intercostal nerve block will accomplish this goal.

Although supplemental oxygen (choice A) would not be directly injurious, it would neither eliminate the pain nor preserve ventilation.

Systemic narcotic analgesics (choice B) would diminish the pain but would also increase the probability of complications by depressing the respiratory drive, thus reducing ventilation.

Binding the chest (choice C) diminishes the pain by limiting motion. In doing so, however, it limits ventilation.

Open reduction and internal fixation to accelerate healing (choice E) is totally unnecessary. The chest wall already is holding the rib in a good position for eventual healing. It will not happen faster if we intervene.

How well did you know this?
1
Not at all
2
3
4
5
Perfectly
8
Q

A 54-year-old woman is brought to the emergency department after a head-on automobile accident. On arrival, she is breathing well. She has multiple bruises over the chest and multiple sites of point tenderness over the ribs. X-ray films show multiple rib fractures on both sides, but the lung parenchyma is clear, and both lungs are expanded. Two days later she is in respiratory distress, and her lungs. “white out” on repeat chest x-ray films. Which of the following is the most likely diagnosis?

(A) Flail chest
(B) Myocardial contusion
(C) Pulmonary contusion
(D) Tension pneumothorax
(E) Traumatic rupture of the aorta

A

Respuesta: C

The correct answer is C. Severe blunt trauma to the chest can produce obvious injuries, such as broken ribs, but it can also lead to pathology that may not show up until later, such as pulmonary contusion or myocardial contusion. The former produces the classic fluid overload, and the fluid leaks easily) along with respiratory “white-out” of the lung (contused lung is exquisitely sensitive to distress.

Flail chest (choice A) is recognized by the paradoxical motion of a segment of the chest wall, which is not described here.

Myocardial contusion (choice B) shows up like an infarction, both clinically (arrhythmias) and on ECG. You would expect it in association with sternal fractures rather than with rib fractures.

Tension pneumothorax (choice D) produces shock and high central venous pressure (CVP), along with the respiratory distress, and air is seen in the x-ray.

The ultimate hidden injury in blunt chest trauma is traumatic rupture of the aorta (choice E). X-ray films would show widening of the mediastinum, and the eventual clinical manifestation would be exsanguinating hemorrhage.

How well did you know this?
1
Not at all
2
3
4
5
Perfectly
9
Q

Renal ultrasound and intravenous pyelography (IVP) in a 65-year- old man evaluated for urinary incontinence reveal bilateral hydronephrosis. Which of the following is the most likely leading to this complication?

(A) Age-associated detrusor overactivity
(B) Alzheimer disease
(C) Normal pressure hydrocephalus
(D) Previous surgery
(E) Prostatic hyperplasia
(F) Stress incontinence

A

Respuesta: E

The correct answer is E. Prostatic hyperplasia results in partial obstruction of the proximal urethra, causing hesitancy and decreased force of stream. With increasing degrees of prostatic enlargement, the volume of urine remaining in the bladder after voiding increases progressively until complete urinary retention manifests with occasional overflow incontinence. Urinary retention leads to dilatation of the ureters and renal pelves (hydronephrosis).

Age-associated detrusor overactivity (choice A) is the most common cause of urinary incontinence in the elderly. It manifests with an uncontrollable urge to urinate not triggered by stress maneuvers. It seems to be related to a deficiency in the descending pathways that inhibit the voiding reflex triggered by bladder distension. This condition does not lead to urinary retention.

Urinary incontinence associated with Alzheimer disease (choice B) and normal pressure hydrocephalus (choice C) is similar to detrusor overactivity and results from failure to inhibit the contractions of the vesical detrusor muscle.

Previous surgery (choice D) may cause sphincteric damage, resulting in total incontinence, in which leakage of urine is continuous. Obviously, this condition will not result in hydronephrosis since there is no obstruction to urinary outflow.

Stress incontinence (choice F) is the second most common cause of urinary incontinence. It is frequent in women and rare in men. It manifests with instantaneous leakage during stress maneuvers, such as coughing. It does not lead to urinary obstruction or retention and thus is not associated with hydronephrosis.

How well did you know this?
1
Not at all
2
3
4
5
Perfectly
10
Q

A 57-year-old man is undergoing a femoral-popliteal bypass of his right lower extremity because of severe peripheral vascular disease. This patient has a longstanding history of claudication and shortness of breath. He had a myocardial infarction 3 years ago and has had progressive limitation of his exercise capacity because of his peripheral vascular disease. He has not had any risk stratification after his infarction. Two weeks ago, he underwent a lower extremity arterial study that showed severe diffuse disease of his right leg arterial system. The patient is brought to the operating room, and, during the procedure, his right lower extremity is made bloodless by application of a thigh tourniquet for 1.5 hours. The surgeons complete their bypass and are preparing to restore blood flow. Which of the following is an expected consequence of this maneuver?

(A) Decrease in blood pressure
(B) Increase in cardiac output
(C) Increase in preload
(D) Increase in venous return
(E) Sinus bradycardia

A

Respuesta: A

The correct answer is A. Vascular surgical patients are often managed by the medical consult service because of the tremendous number of co-morbidities. During vascular procedures, the use of cross-clamping and tourniquets produces localized or regional ischemia. The consequences of ischemia include the accumulation of metabolic waste products and acid load (so-called evil humors), which are freely available to wreak havoc on the systemic circulation once they gain access to it. The primary consequence of this is profound and dramatic systemic hypotension that can be prolonged for hours after a procedure. Such a phenomenon has obvious consequences for management of patients such as this man with coexisting cardiac disease.

A drop in systemic blood pressure from severe vasodilatation will lead to decreased preload (compare with choice C) and thus a decrease in stroke volume and cardiac output (choice B).

Restoration of circulation to the previously clamped limb opens an entirely new venous reservoir, thus dramatically reducing venous return (compare with choice D). In addition, the massive systemic vasodilatation would further decrease venous return. The result of these two events is a dramatic, and often profound, drop in systemic blood pressure.

Sinus bradycardia (choice E) is the opposite of the reflex tachycardia that is expected with profound hypotension.

How well did you know this?
1
Not at all
2
3
4
5
Perfectly
11
Q

A 31-year-old man is brought to the emergency department after a motor vehicle accident. He sustained a severe head injury and, on arrival to the emergency department, has a Glasgow coma score of 8. His blood pressure is stable, and an urgent CT scan of the head reveals a large subdural bleed with evidence of a midline shift and cerebellar tonsillar compression. The patient is breathing spontaneously without any respiratory assistance and is not intubated. Which of the following is the most appropriate next step in management?

(A) Obtain an urgent head MRI to evaluate for herniation
(B) Administer IV mannitol
(C) Perform endotracheal intubation and hyperventilation
(D) Induce a barbiturate coma
(E) Initiate immediate surgical decompression

A

Respuesta: C

The correct answer is C. This patient has an intracranial bleed, signs of increased intracranial pressure (ICP), and evidence on a CT scan of impending herniation. This patient requires rapid lowering of his ICP. The most rapid method available is hyperventilation to lower PaCO2, which leads to decreased cerebral blood flow and ICP.

Obtaining an urgent head MRI to evaluate for herniation (choice A) is unnecessary since the head CT already showed clear signs of impending herniation. An MR scan adds nothing to the decision analysis and need for immediate therapy.

Administration of IV mannitol (choice B) is also an appropriate therapy in this case. However, mannitol has an onset of action approximately 90 minutes after dosing, which makes hyperventilation the mainstay of acute therapy.

Induction of a barbiturate coma (choice D) is used as a last resort to dramatically lower ICP. In cases of severe emergency, patients are mechanically ventilated and placed in a barbiturate coma so that maximal lowering of ICP can be attained.

Initiating immediate surgical decompression (choice E) may be appropriate, but not until hyperventilation has begun. Like mannitol, surgical decompression (even as emergent surgery) is not immediate; therefore, therapy needs to be instituted during that interval.

How well did you know this?
1
Not at all
2
3
4
5
Perfectly
12
Q

A 40-year-old retired professional football player complains of the sudden onset of palpitations and shortness of breath 5 days after having knee replacement surgery. His pulse is 100/min and regular. Oxygen saturation is 90% room air. An ECG reveals sinus tachycardia. A chest x-ray film is unremarkable. Which of the following is the most appropriate next step in management?

(A) Order an arterial blood gas
(B) Schedule a duplex Doppler examination of the lower extremities
(C) Schedule a ventilation-perfusion scan
(D) Administer supplemental oxygen
(E) Administer IV heparin

A

Respuesta: A

The correct answer is A. This patient most likely has a pulmonary embolism. Pulmonary embolism occurs following general surgery in 1 to 2% of patients older than 40. The incidence is higher (5 to 10%) following ortho-pedic surgery of the hip or knee. Venous stasis due to immobilization is probably a major reason for venous thrombosis associated with surgery. However, other factors, such as increased blood fibrinolytic activity and vessel damage, may be involved as well. An increased alveolar-arterial oxygen difference (A-a gradient) seen on arterial blood gas on room air supports the diagnosis, along with sinus tachycardia on ECG and a normal chest x-ray.

Duplex Doppler examination (choice B) and ventilation-perfusion scan (choice C) are important in confirming the diagnosis of pulmonary embolism, but an arterial blood gas should be performed first.

Supplemental oxygen (choice D) should be given after an arterial blood gas is obtained.

IV heparin (choice E) is the treatment of choice for a pulmonary embolus but should not be administered until there are some objective data that support the diagnosis.

How well did you know this?
1
Not at all
2
3
4
5
Perfectly
13
Q

A 19-year-old man is involved in a motorcycle accident in which he sustains a closed fracture of his right femur and a pelvic fracture. In addition to the obvious deformity in his leg, physical examination is remarkable for the presence of a scrotal hematoma and blood at the meatus. There is no blood in the rectal exam, but the prostate cannot be felt. The patient states that he feels the need to void, but cannot do it. Which of the following is the most appropriate next step in diagnosis?

(A) CT scan of the pelvis
(B) Scrotal sonogram
(C) IV pyelogram (IVP)
(D) Retrograde cystogram via Foley catheter
(E) Retrograde urethrogram

A

Respuesta: E

The correct answer is E. The hallmark of a urologic injury is a trauma patient who has blood in the urine (or in the visible part of the urinary tract, as in this case). When a pelvic fracture is also present, we have to bet on the lower urinary tract: the bladder in either gender, or the bladder or urethra in the male. When the blood is visible at the meatus and you add the scrotal hematoma, the wall: urethral injury. The last thing you want to do in this case is “vanishing” prostate, and the inability to void, the writing is on the insert a Foley catheter; you might convert a partial urethral disruption into a complete transection. You want to inject the dye directly into the urethra (retrograde urethrogram).

A CT scan of the pelvis (choice A) might be needed to assess pelvic bleeding (if we had been told that this man was in shock), but it would not be the best way to detect a urethral leak.

A scrotal sonogram (choice B) can tell you whether the testicle is injured, but a ruptured testicle does not give you blood in the urinary tract.

The IV pyelogram (choice C) would be a round-about and unreliable way to get radiopaque material where you need it.

As pointed out above, inserting a Foley catheter to do a cystogram (choice D) would be absolutely contraindicated if the clinical picture suggests urethral injury.

How well did you know this?
1
Not at all
2
3
4
5
Perfectly
14
Q

A 25-year-old man is shot with a .22-caliber revolver. The entrance wound is in the anterior, lateral aspect of his thigh, and the bullet is seen on x-ray films to be embedded in the muscles posterolateral to the femur. The emergency department physician cleans the wound thoroughly. Which of the following is the most appropriate next step in management?

(A) Tetanus prophylaxis
(B) Doppler studies
(C) Arteriogram
(D) Surgical exploration of the femoral vessels
(E) Surgical removal of the embedded bullet

A

Respuesta: A

The correct answer is A. All penetrating injuries require tetanus prophylaxis, an often overlooked detail when dealing with other more impressive problems. In this case, the key to the correct answer lies in the fact that the other options are not indicated. In gunshot wounds of the extremities, the main concern is the possibility of major vascular injuries.

Such injuries can be evaluated with Doppler studies (choice B), arteriograms (choice C), or surgical exploration (choice D), but none of those are needed here. A rudimentary knowledge of anatomy allows the physician to skip all those expensive procedures: the femoral artery (with the femoral vein adjacent to it) is located anteromedial in the upper thigh and eventually becomes centered on the axis of the extremity when it becomes the popliteal. It is never located on the lateral side of the thigh, where the bullet tract is located in this vignette.

Removing the bullet (choice E), although obligatory in Western movies, is not necessary if it is not threatening to erode some vital structure.

How well did you know this?
1
Not at all
2
3
4
5
Perfectly
15
Q

A patient sustained third-degree burns on both his arms when his shirt caught on fire while he was lighting the backyard barbecue. The burned areas are dry, white, leathery, anesthetic, and circumferential all around the arms and forearms. Which of the following parameters should be very closely monitored?

(A) Blood gases
(B) Body weight
(C) Carboxyhemoglobin levels
(D) Myoglobinemia and myoglobinuria
(E) Peripheral pulses and capillary filling

A

Respuesta: E

The correct answer is E. Circumferential burns of the extremities pose a distinct hazard to peripheral circulation because the edema fluid resulting from the burn cannot expand under the unyielding envelope of the burn eschar. Compulsive monitoring of pulses and capillary filling is required; escharotomy also may be required.

Although flame burns can cause smoke inhalation and the so-called respiratory burn, they do so only when the victim is trapped in an enclosed space: a burning car, a plane, a building. In those situations you would monitor blood gases (choice A) and carboxyhemoglobin (choice C) levels. This fellow was burned in the backyard (a well- ventilated place), so these are not a valid concern.

Body weight (choice B) does not change much with the massive internal fluid shifts of a major burn. We guide our fluid therapy by urinary output and central venous pressure, not by monitoring body weight.

As for myoglobinemia and myoglobinuria (choice D), they are of paramount concern in high voltage electrical burns or crushing injuries, not in flame burns.

How well did you know this?
1
Not at all
2
3
4
5
Perfectly
16
Q

A previously healthy 60-year-old man is referred for urologic evaluation of macroscopic hematuria. Urinary cytology is positive for malignant cells, and cystoscopic examination reveals an exophytic multifocal tumor. A biopsy of the tumor demonstrates papillary fronds lined by cells similar to transitional epithelium but showing nuclear atypia, mitoses, and necrosis. Which of the following is the most important risk factor in the U.S. for the development of this type of tumor?

(A) Aniline dyes
(B) Cyclophosphamide
(C) Phenacetin
(D) Radiation
(E) Recurrent cystitis
(F) Schistosomiasis
(G) Smoking

A

Respuesta: G

The correct answer is G. Transitional cell tumors represent 90% of the neoplasms arising from the urinary bladder. These tumors grow as exophytic, papillary masses, which most commonly present with gross or microscopic hematuria. Cigarette smoking is epidemiologically the most significant risk factor for transitional cell carcinoma and seems to account for more than two thirds of cases. Formation of carcinogenic polycyclic aromatic hydrocarbons derived from tobacco smoke plays a key role in the pathogenesis.

Persons with a prolonged occupational exposure to aniline dyes (choice A), particularly beta-naphthylamine, exhibit a 50-fold increase in the incidence of bladder cancer of the transitional type compared with the general population. Currently, prior exposure to aniline dyes accounts for approximately 15% of the cases of bladder cancer.

The immunosuppressant drug cyclophosphamide (choice B) and the analgesic phenacetin (choice C) increase the likelihood of developing transitional cell carcinomas of the urinary tract, but cases related to these drugs are rare.

No relationship has been shown between transitional cell carcinoma of the bladder and radiation (choice D) or recurrent cystitis (choice E).

Urinary schistosomiasis (choice F), due to infestation with Schistosoma hematobium, is the most significant important risk factor for urinary bladder cancer in countries such as Egypt and Sudan, where schistosomiasis is endemic. Most of these tumors, however, are squamous (not transitional) cell carcinomas.

How well did you know this?
1
Not at all
2
3
4
5
Perfectly
17
Q

A 23-year-old man is admitted to the hospital after being struck by a motor vehicle. The patient sustained a compound fracture of his left femur in the accident and has had moderate blood loss. He was admitted to the hospital, has been stabilized over the past few days, and is now preparing for physical therapy. His hematocrit is 24%. The man feels weak and fatigued and easily gets short of breath with mild exertion. Which of the following is the most appropriate next step in management?

(A) Continue with physical therapy; no transfusion is indicated
(B) Discontinue physical therapy until the patient recovers more of his strength
(C) Transfuse fresh frozen plasma to a hematocrit goal of 30%
(D) Transfuse packed red blood cells to a hematocrit goal of 30%
(E) Transfuse whole blood to a goal hematocrit of 30%

A

Respuesta: D

The correct answer is D. Transfusion of packed red cells [preparation of all the red cell mass from a pint of donated blood-it has no plasma or buffy coat and therefore no proteins (coagulation factors) or platelets] is one of the most frequent treatments executed by physicians. There exists in medicine a dogma of uncertain origin that states that anemic patients with a hematocrit less than 30% should be transfused with red cells until that value is attained. This rule is even more rigidly followed in patients with co-existing illness, such as cardiac or pulmonary disease.

Although this “rule” is being called into question as incorrect by recent publications, it is still generally accepted that patients with acute bleeds, such as the one in this vignette, merit repletion of red cells if they are symptomatic from such a bleed. Continuing with physical therapy without transfusion (choice A) is a choice favored by many physicians. This is because many people believe that a 23-year-old man will replete his own red cells over time. In this case, however, the patient is clearly symptomatic with even minimal exertion. Therefore, his anemia is not benign and merits treatment. There is, of course, no reason to restore his pre- accident hematocrit, but he should be transfused to a level at which his symptoms would be lessened or abrogated (about 30%).

Discontinuing physical therapy until the patient recovers more of his strength (choice B) is not appropriate since the patient requires therapy to regain his strength, and the reason for his weakness likely relates to his acute anemia.

Transfusion of fresh frozen plasma (FFP) to a hematocrit goal of 30% (choice C) is incorrect. FFP is used to restore clotting factors. One unit generally increases plasma anticoagulation factors by 30%. Like all blood products, it is type-specific.

Transfusion of whole blood to a goal hematocrit of 30% (choice E) is not performed. Whole blood is the content of 1 pint of donated blood. It is unfiltered and contains plasma, platelets, white cells, and red cells. This product is usually processed so that each of these components are removed (except white cells) and used for transfusions in specific clinical situations.

How well did you know this?
1
Not at all
2
3
4
5
Perfectly
18
Q

A 24-year-old man comes to the physician 24 hours after sustaining an injury to the right knee while playing soccer. He can walk, but he limps on the right side. He reports that he was hit by another player on the lateral side of his right knee, but did not feel a snap or pop at the time of the accident. On examination, the right knee appears normal, but palpation elicits tenderness along the medial aspect of the joint line. Increased laxity is observed when a valgus stress is applied to the knee flexed at 30 degrees, but not when the knee is in full extension. Lachman’s test and posterior drawer tests are negative. Which of the following is the most likely diagnosis?

(A) Meniscus injury
(B) Sprain of the lateral collateral ligament
(C) Sprain of the medial collateral ligament
(D) Tear of the anterior cruciate ligament
(E) Tear of the posterior cruciate ligament

A

Respuesta: C

The correct answer is C. The patient presents with the typical symptomatology associated with sprain of the medial collateral ligament. This ligament connects the distal femur to the proximal tibia on their medial aspects. Injuries to this ligament are the most frequent among traumatic knee injuries and typically result from a lateral blow to the joint. The injured knee is sometimes swollen, but often inspection reveals only walking difficulties. Physical examination should include maneuvers that assess ligamentous stability, comparing the injured and uninjured sites. A valgus stress test demonstrating increased laxity of the knee confirms sprain of the medial collateral ligament. These tests should be performed on both flexed and extended knees. Increased laxity of ligaments with the knee in full extension indicates concomitant capsular injury (absent in this case).

Meniscus injury (choice A) often results in a “locked-up” knee and is usually due to traumas that have a twisting component. Appropriate tests to evaluate meniscal integrity (such as the McMurray) should be part of the physical examination in case of knee injuries.

Sprain of the lateral collateral ligament (choice B) is usually due to blows to the medial aspect of the knee. The varus stress test would be positive.

Tears of the anterior cruciate ligament (choice D) and posterior cruciate ligament (choice E) will result in knee instability. Often, the patient reports feeling a snap or pop at the time of injury. Lachman’s test is the most sensitive clinical maneuver to detect injuries to the anterior cruciate ligament. The examiner stabilizes the knee with one hand and pulls the tibia forward. Any forward movement of the tibia (compared with the uninjured side) is considered diagnostic of anterior cruciate ligament tear. The posterior drawer test is used to detect tears of the posterior cruciate ligament.

How well did you know this?
1
Not at all
2
3
4
5
Perfectly
19
Q

A 48-year-old man with alcoholic cirrhosis has several episodes of massive hematemesis. Upper gastrointestinal endoscopy confirms that he is bleeding from esophageal varices. Sclerosing injections fail to control the bleeding. After the patient has been transfused 7 units of packed red cells, he is subjected to an emergency side-to- side portacaval shunt. At the time of surgery he has a serum albumin level of 3.1 g/dL, a total bilirubin of 1.7 mg/dL, and a prothrombin time (PT) 2 seconds above the control. After surgery, the bleeding stops, and the patient wakes up briefly from the anesthetic but then lapses into a coma. The reason for his neurologic deterioration would most likely be revealed by a laboratory determination of which of the following?

(A) Blood alcohol levels
(B) Blood gases
(C) Blood glucose
(D) Serum ammonia
(E) Serum sodium

A

Respuesta: C

The correct answer is D. Portacaval shunts are very effective in decreasing the pressure in esophageal varices, and thus controlling bleeding from them. But the penalty paid for that diversion of blood flow is further impairment of liver function. One almost never sees cirrhotic patients come to surgery with normal liver function. And, if they are bleeding at the time, they also have a load of ammonia in the gut that has to be cleared by the liver. With the initial limited function, plus the trauma of surgery and the diversion of portal flow, ammonia (as well as other toxic substances) accumulates in the blood and leads to coma.

Blood alcohol levels (choice A) would be relevant in an alcoholic who has been drinking up to the time that some unexpected event necessitates emergency surgery. If the patient comes to the operating room with high levels of alcohol in the blood, one can predict that delirium tremens (DTs) will occur 2 or 3 days later.

Determination of blood gases (choice B) is always the first thing to do when unexplained mental deterioration occurs after surgery. Hypoxia is very likely to be the culprit. In this case, however, we do not have an unexplained occurrence, but one rather predictable problem.

Blood glucose (choice C) comes to mind for the diabetic patient known to use insulin who suddenly goes into coma, or for the unknown patient brought to the emergency department in coma and with no history of what happened to him. Although it is true that hypoglycemia is seen in liver failure, it occurs at the very end of the spectrum, when all other parameters of liver function are grossly deranged.

Rapid changes in serum sodium (choice E) can cause coma, such as in the precipitous hyponatremia seen in water intoxication or the hypernatremia of profound dehydration. Neither of those are likely to occur, however, in the setting of this vignette.

How well did you know this?
1
Not at all
2
3
4
5
Perfectly
20
Q

A 57-year-old man is returned to the post-surgical recovery unit after an open cholecystectomy. The patient had an uneventful, but prolonged, operative course in a very cold operating room. His past medical history is unremarkable. The only attempt at patient warming was raising the ambient temperature of the room. His urine output since arrival in the post-anesthesia care unit (PACU) has been 5 mL/hr. Which of the following is most likely to confirm the diagnosis?

(A) Low serum aldosterone
(B) Serum BUN to creatinine ratio greater than 20
(C) Urine osmolality of 280 mOsmol/kg
(D) Urine sodium of 40 mEq/L
(E) Urine specific gravity of less than 1.010

A

Respuesta: B

The correct answer is B. Post-surgical patients generally have moderate to severe derangement in fluid balance. They have been fasted before the procedure and then had a variety of sensible and insensible losses during the procedure. In this case, the idea of severe dehydration causing prerenal azotemia would be supported by an elevated BUN and creatinine, but in a ratio of greater than 20:1. This is due to the heightened reabsorption and retention of solute by the kidney that is reflected by the elevated BUN.

Low serum aldosterone (choice A) is incorrect. In conditions of volume depletion, the renin-angiotensin-aldosterone axis is activated with high levels of each hormone. In this case, aldosterone is acting on the distal tubules to affect sodium reabsorption.

Urine osmolality of 280 mOsmol/kg (choice C) is incorrect because in the case of volume depletion, the urine should be maximally or near maximally concentrated, reflecting retention of nearly all filtered water.

A urine sodium of 40 mEq/L (choice D) is not correct. With volume depletion, the urine sodium should be quite low (< 20 mEq/L), reflecting retention of nearly all filtered water and sodium.

A urine specific gravity of less than 1.010 (choice E) is the opposite of what is expected. As with osmolality, this parameter should reflect maximal concentration of the urine, which is equivalent to minimal free water excretion.

How well did you know this?
1
Not at all
2
3
4
5
Perfectly
21
Q

A 25-year-old man presents to the same day surgical center for repair of an old injury to his lateral collateral ligament. The anesthesiologist wants to perform an axillary block for local pain control. If the posterior wall of the axillary artery is pierced during placement of the block, which of the following nerves will most likely be affected?

(A) Axillary
(B) Median
(C) Musculocutaneus
(D) Radial
(E) Ulnar

A

Respuesta: E

The correct answer is E. This question simply requires a basic understanding of the anatomy of the brachial plexus. In every medical specialty, general medicine included, knowledge of key anatomic loci is crucial for patient care. Classic examples of this include placement of central venous lines or needle thoracentesis. In this case, the ulnar nerve, the end-terminal branch of the medial cord (posterior to the axillary artery) of the brachial plexus, is in jeopardy. Although the posterior cord is posterior to the axillary artery at lower levels, at this level the medial cord is interposed between the posterior cord and the artery. This is the so-called “region two of the axillary artery (posterior to the pectoralis minor muscle),” where the axillary block is performed.

The axillary nerve (choice A) is a branch of the posterior cord, but arises very high in the plexus and immediately exits the axilla via the teres muscle groups.

The median nerve (choice B) is formed from the medial and the lateral cords, is very low in the brachium, and is not in danger from an axillary block.

The musculocutaneus nerve (choice C) is a branch of the lateral cord and is in no danger, as it is buried in muscle tissue from its origin.

The radial nerve (choice D), also a branch of the posterior cord, is in no danger of injury since it exits the axilla via the radial groove on the humerus, very deep to muscle. This nerve is most often injured during spiral fractures of the humerus.

22
Q

A 49-year-old woman seeks help for a vague, constant, epigastric distress that she began experiencing about 5 weeks after returning from a 10-day trip to Mexico. She relates that she drove to Mexico City and Guadalajara and was very careful with what she ate and drank. Nevertheless, she experienced acute diarrhea on the third day of her trip and was treated by a hotel physician with a pharmaceutical product that was said to contain “locally acting antibiotics.” She had no further gastrointestinal complaints, but on her drive home, she was involved in an automobile accident. She hit a cow that was crossing the road and suffered epigastric trauma when her upper abdomen hit against the steering wheel. She was kept overnight at a hospital in Monterrey for clinical observation and was discharged the next morning. She did not seek further medical help when she got to the United States because she was asymptomatic. On physical examination, she has a deep, large, ill- defined, epigastric mass that is not tender to palpation. She is afebrile, and her only other complaint is that she cannot eat a full meal because she feels “full” right away. After confirmation of the suspected diagnosis, the treatment of her condition may require which of the following?

(A) Deployment of an intra-arterial stent
(B) Endoscopic anastomosis
(C) Laparoscopic repair of the injured structure
(D) Long-term antibiotic therapy
(E) Resection of the affected part of the liver

A

Respuesta: B

The correct answer is B. Vague, epigastric distress, early satiety, and a large but ill-defined, epigastric mass developing 5 weeks after trauma to the upper abdomen is one of the classic presentations of a pancreatic pseudocyst (the other presentation would follow an episode of pancreatitis). Small, pancreatic pseudocysts may go away during clinical observation, but big, palpable pseudocysts probably will not. Thus, the patient will probably require either internal or external drainage. The most sophisticated way to achieve drainage is by performing an endoscopic cystogastrostomy. Two older treatments, radiologically guided external drainage and surgically constructed internal derivation, were not offered as options.

Intra-arterial stents (choice A) are used in conjunction with angioplasty for vascular stenosis, or as an experimental treatment for abdominal aortic aneurysms. This woman does not have an aneurysm. Aneurysms of the abdominal aorta are not produced by trauma, do not interfere with eating, and are clearly palpable as a pulsatile mass.

Assuming that you had the correct diagnosis-a pancreatic injury that resulted in a pseudocyst-laparoscopic repair (choice C) is not the correct treatment. We do not attempt to repair the pancreas to treat this condition. We simply reroute the pancreatic juice, and, eventually, the injury heals by itself.

The exposure to exotic bugs in Mexico and the brief episode of diarrhea might have led you into thinking of an infection that might require antibiotics (choice D). However, no such infection would produce a deep, epigastric mass, and the patient is afebrile. A brief episode of diarrhea is very common for travelers; it does not necessarily lead to further pathology. In this case, it is simply a red herring.

At the same time, someone who becomes ill with diarrhea after visiting Mexico could be thought of as having an amebic abscess of the liver. But such patients are febrile, and their livers are tender. Their treatment starts with metronidazole and may require drainage but not resection of the affected part of the liver (choice E).

23
Q

A 35-year-old man had a splenectomy 8 days ago, following a motor vehicle accident. He is now complaining of left shoulder pain. His temperature is 39.0 C (102.2 F), blood pressure is 110/80 mm Hg, pulse is 110/min, and respirations are 30 min and shallow. Physical examination shows clear lungs with equal breath sounds bilaterally and mild tenderness to palpation in the left upper quadrant with a well-healing midline laparotomy incision. Laboratory studies show:

Hemoglobin 15 g/dL
Hematocrit 45%
Leukocyte counts 15,000/mm3

A chest x-ray film shows no infiltrates or effusions. Which of the following is the most likely diagnosis?

(A) Left clavicle fracture
(B) Left lower lobe pneumonia
(C) Post-splenectomy sepsis
(D) Subphrenic abscess
(E) Subphrenic hematoma

A

Respuesta: D

The correct answer is D. Subphrenic abscess is a common complication of splenectomy and is implied by the patient’s elevated temperature and elevated WBC, pleuritic pain (which is the probable cause of his rapid and shallow respirations), and left upper quadrant tenderness. A subphrenic abscess would irritate the phrenic nerve (nerve root C3-C5), causing referred pain toward dermatome of the nerve root, which includes the left shoulder.

Left clavicular fractures (choice A) appear erythematous at the site of fracture and exhibit crepitus on palpation. The arm is usually held close to the body, and the ipsilateral shoulder appears lower than the opposite side.

One would expect rales or rhonchi instead of clear lungs and equal breath sounds in a patient with left lower lobe pneumonia (choice B).

Post-splenectomy sepsis (choice C) would not produce such localized symptoms.

Subphrenic hematoma (choice E) is not consistent with the fever and leukocytosis observed in this patient.

24
Q

A 24-year-old woman is brought to the emergency department after being stabbed by her boyfriend. The examining physician notes a 1.5-cm puncture wound lateral to her sternum. She has a blood pressure of 70/palpable, distended neck veins, and muffled heart sounds. Which of the following is the most appropriate next step in management?

(A) Cardiac surgery consult
(B) Echocardiogram
(C) Chest x-ray film
(D) Chest tube placement
(E) Pericardiocentesis

A

Respuesta: E

The correct answer is E. The woman was stabbed in the heart, leading to cardiac tamponade (blood collecting in the pericardial sac). This causes impairment in heart function, leading to hypotension, distension of neck veins due to pump failure, and muffled heart sounds due to the collection of blood. The immediate concern is removing the blood from the pericardial sac by performing pericardiocentesis. All the other tests would lead to unnecessary delays in diagnosis and would result in death.

A cardiac surgery consult (choice A) is necessary for this patient to ultimately repair the damaged heart; however, the first step in saving this woman before the specialist arrives is pericardiocentesis.

Echocardiogram (choice B) could aid in the diagnosis of pericardial effusion but would take too long to administer in such an emergent situation.

A chest x-ray film (choice C) would show a pericardial effusion, but there already are enough data to support the diagnosis, so x-ray would cause unnecessary delay in therapy.

Chest tube placement (choice D) is used for pneumothorax and pleural effusions but would not be effective in the present scenario.

25
A 55-year-old-woman of Asian descent goes to the emergency department because of vomiting and severe abdominal cramping of 3 days’ duration. Her pain is centered on the umbilicus. She denies being exposed to a viral or bacterial illness. Her medical history includes a previous cholecystectomy and an appendectomy after which she developed an infection. Her abdomen is not tender, but hyperactive, high-pitched peristalsis with rushes coincides with palpable bowel cramping. Abdominal x-ray films taken in the supine and upright positions demonstrate a ladder-like series of distended small bowel loops. Which of the following is the most likely explanation for these findings? (A) Adhesions (B) Ascaris infection (C) Cancer (D) Intussusception (E) Volvulus
Respuesta: A The **correct answer is A**. The clinical and radiologic findings are typical of small bowel (jejunoileal) obstruction. If the pain becomes continuous and bowel sounds vanish, suspect that strangulation has occurred. The most frequent causes of small bowel obstruction are adhesions and incarceration in hernias. To investigate adhesions, look for a history of peritonitis, Crohn disease, pelvic inflammations, or abdominal or pelvic surgery. Overwhelming Ascaris infection **(choice B)** can cause small bowel obstruction, most frequently in poorly developed, third-world tropical countries. Cancer **(choice C)** of the duodenum or pancreas is the most common cause of obstruction of the duodenum, but the ladder-like series of dilated bowel loops would not be seen because the obstruction would be proximal to the ileum. Intussusception **(choice D)** is relatively common in infants but is much rarer (and usually related to tumor) in adults. Volvulus **(choice E)** is rare in the midgut.
26
A young man is shot in the upper part of the neck with a .22 caliber revolver. Inspection of the entrance and exit wounds indicates that the trajectory of the bullet is all above the level of the angle of the mandible, but below the skull. He is fully conscious and neurologically intact. A steady trickle of blood flows from both wounds, and it does not seem to respond to local pressure. He is hemodynamically stable. Which of the following is the most appropriate next step in diagnosis? (A) Continued clinical observation (B) Barium swallow (C) Arteriogram (D) Endoscopy (E) Surgical exploration
Respuesta: C The **correct answer is C**. In gunshot wounds of the upper zone of the neck, the main concern is the possibility of significant vascular injuries. The area is too high to involve the aerodigestive tract, and it is also rather difficult to explore surgically. Arteriogram offers the best way to assess the extent of the injuries, and also provides a way for embolization of major arteries that might be bleeding significantly. Clinical observation **(choice A)** is the second best answer, but it would delay recognition of significant vascular injuries that the arteriogram might demonstrate. Clinical observation is often all we do in asymptomatic stab wounds, where serious damage is less likely to occur. Barium studies **(choice B)** are essential when one suspects esophageal injury that is not demonstrated by gastrografin swallow. As pointed out above, however, the area of injury here is well above where the esophagus begins. Endoscopy **(choice D)** is incorrect for the same reasons that barium studies have no role in this case: the trajectory of the bullet is too high to involve the aerodigestive tract. Surgical exploration **(choice E)** might be unavoidable in hemodynamically unstable patients whose vascular injuries cannot be controlled by arteriographic embolization. Surgery can be performed in this area if needed, but for technical reasons it is not our first choice of management.
27
A 60-year-old woman has a lumpectomy and a sentinel node biopsy performed for an infiltrating ductal carcinoma on the upper outer quadrant of her right breast. The surgical specimen measures 12 by 10 by 8 cm, and all of the surgical margins are reported as negative by the pathologist. The aggregate of the measurements and studies done on the specimen reveals that the size of the tumor was 3.8 by 3.5 by 2.8 cm. Two sentinel axillary nodes removed by the surgeon are negative for metastasis. The tumor is strongly positive for estrogen and progesterone receptors. Histologic grade was III/III, and prognostic studies of flow cytometric S phase, DNA index, ploidy, and Ki-67 antigen are all reported as unfavorable. Prior to surgery, the patient agrees to receive postoperative radiation therapy for the right breast. Which of the following would optimize her chances for a cure? (A) Chemotherapy and anastrozole (B) Chemotherapy and tamoxifen (C) Chemotherapy plus estrogens and progesterone (D) Completion of the operation to a radical mastectomy (E) Radiation therapy to the contralateral breast and the right axilla
Respuesta: A The **correct answer is A**. There are two favorable findings in this patient: Her axillary lymph nodes do not have metastasis, and her tumor is strongly positive for hormonal receptors. However, everything else is unfavorable: The tumor is quite large, and the prognostic factors are all unfavorable. Clearly, the patient would benefit from chemotherapy, and she should be placed on hormonal therapy after chemotherapy and radiation therapy to take advantage of the positive hormonal receptors. The objective of the hormonal manipulations is to either block her receptors (with tamoxifen) or suppress the production of estrogens (with anastrozole). Randomized studies have shown that anastrozole is considerably more effective than tamoxifen in postmenopausal patients. Thus, it has become the drug of choice for that group. Chemotherapy and tamoxifen **(choice B)** would be the best course of action for premenopausal patients, for whom anastrozole is not yet approved as the best drug. Chemotherapy and tamoxifen would have been the regimen for this patient before the advent of anastrozole. Adding estrogens and progesterone to the chemotherapy **(choice C)** would be contraindicated. We want to deprive the tumor of the stimulation provided by those hormones. Completion of the operation to a more radical form of resection **(choice D)** has no benefits. The patient has a very large margin of normal tissue around the tumor, all the margins are negative, and the patient has agreed to postoperative radiation. We do not cure breast cancer by cutting out more normal tissue. We do it by treating the systemic spread of the disease. Radiation therapy **(choice E)** is focal in nature. It does not treat distant metastatic spread. The patient needs radiation to the breast that had the tumor to minimize the rate of local recurrence. Radiating the other breast and the axilla would not be helpful.
28
A 62-year-old woman faints while waiting in line to get into a movie theater. When she is examined by her physician the next day, she is found to be pale, with yellowish sclera, and to have a hemoglobin level of 7 g/dL. Except for mild obesity, the rest of the physical examination is unremarkable, but her stool is strongly positive for occult blood. She is told that she will need a colonoscopy, but before the study is done, further laboratory results become available, showing that she has a total bilirubin of 3.5 mg/dL and an alkaline phosphatase of 850 U/L. The transaminases are minimally elevated. Her physician then orders a sonogram of the right upper quadrant, and the study shows dilated intrahepatic ducts, dilated extrahepatic ducts, and a large, distended, thinwalled gallbladder without stones. Which of the following should be the next diagnostic study performed? (A) Barium enema (B) CT scan of the liver (C) Liver biopsy (D) Percutaneous transhepatic cholangiogram (E) Upper gastrointestinal endoscopy
Respuesta: E The **correct answer is E**. A woman of this age who is found to be anemic and have occult blood in the stool should be suspected of having a cancer on the right side of the colon. Thus, the colonoscopy that was initially planned was the appropriate study. However, the patient also has obstructive jaundice, which is most likely of malignant origin (she has the sonographic equivalent of a positive Courvoisier-Terrier sign). Good clinical thinking involves attempting to find one single disease that explains all of the concurrent findings. Therefore, assuming that the patient has colon cancer plus a cancer of the head of the pancreas is not a good bet. A single tumor that would bleed into the gastrointestinal lumen and obstruct the common duct is ampullary carcinoma, and upper gastrointestinal endoscopy should identify it. If we insist in ruling out colon cancer, colonoscopy could be done during the session in which upper gastrointestinal endoscopy is performed. Barium enema **(choice A)** would be a less attractive alternative. The problem is not in the liver and, thus, a CT scan **(choice B)** or a biopsy **(choice C)** of the organ is a misguided effort. Dilated intrahepatic ducts and dilated extrahepatic ducts point to a low obstruction of the biliary tree. They are not suggestive of liver metastasis or liver pathology. The laboratory findings are also suggestive of biliary obstruction rather than hepatocellular disease. Further definition of the nature of biliary obstruction can be obtained with an endoscopic retrograde cholangio-pancreatogram (ERCP) or a percutaneous transhepatic cholangiogram **(PTC; choice D)**. However, if we suspect that the obstructing tumor grows out of the duodenal wall, performing the study via a liver puncture (i.e., PTC) will miss the pathology. ERCP would find it, but, in fact, the full study would not have to be completed: As soon as the endoscopist looks at the ampulla, the tumor will be discovered. Cannulation of the ducts and injection of dye would not be needed.
29
A 44-year-old woman has a 2-cm firm palpable mass in the upper outer quadrant of her right breast. The mass is freely movable, and her breast is of normal, rather generous size. There are no palpable axillary nodes. Mammogram shows no other lesions. A core biopsy establishes a diagnosis of infiltrating ductal carcinoma. She has no neurologic or skeletal symptoms, and a chest x-ray film and liver enzymes are normal. She understands that systemic therapy may eventually be needed once the full extent of her disease is known. Although she wants the best chance for cure, she is very concerned about cosmetic deformity and wants to know what can be done about the breast itself. Which of the following is the most appropriate management? (A) Radiation and chemotherapy without breast surgery (B) Lumpectomy, axillary sampling, and postoperative radiation (C) Simple total subcutaneous mastectomy with implants (D) Modified radical mastectomy with immediate rectus abdominis flap reconstruction (E) Radical mastectomy and postoperative radiation, with delayed reconstruction
Respuesta: B The **correct answer is B**. This is actually the ideal candidate for breast-sparing surgery: a patient with a small primary tumor in a large breast, located far away from the nipple and areola. Provided radiation is done afterward, the cure rates are identical to those for more mutilating procedures. The cosmetic outcome is excellent, and no reconstruction is needed (the void left by the lumpectomy fills in with body fluids and is eventually replaced by connective tissue). No surgery at all **(choice A)** is not an option. As much as we want to preserve the breast, and as much as we rely on postoperative radiation to lower the local recurrence rates, leaving the primary tumor in place does not lead to cure. Simple mastectomy **(choice C)** entails more surgery than needed for the tumor (for which a lumpectomy followed by radiation is sufficient in this case), but not enough to learn about the status of the axillary nodes. They have to be sampled (either by dissection or sentinel node biopsy). Physical examination is totally unreliable for that purpose. Although modified radical mastectomy **(choice D)** may be unavoidable in patients with larger primary tumors in smaller breasts, or tumors located where the nipple and areola can’t be preserved, this patient does not need that larger operation (no survival advantage) and shouldn’t take the more complicated and less pleasing breast reconstruction. Old-fashioned radical mastectomy **(choice E)** is unnecessarily aggressive and not justified unless the tumor is huge and invading the pectoralis muscle. Unless surgical margins are positive for tumor, postoperative radiation would be equally unnecessary if the whole breast is taken.
30
In the first postoperative day after an open abdominal procedure, a patient develops a temperature of 38.9 C (102 F). He is encouraged to ambulate, cough, and breathe deeply, but he is noncompliant. On the second day, he is still febrile. Incentive spirometry and postural drainage are instituted, but his participation is less than enthusiastic. He lies in bed all day and hardly moves. By the third day, he is still spiking fevers in the same range. Although efforts to improve his ventilation continue, resolution of his problem will most likely require which of the following? (A) Doppler studies of deep leg and pelvic veins (B) Urinalysis, urinary cultures, and appropriate antibiotics (C) Chest x-ray, sputum cultures, and appropriate antibiotics (D) Cultures of his wound and wound opening if needed (E) CT scan of the abdomen and percutaneous drainage of abscess
Respuesta: C The **correct answer is C**. Fever on the first postoperative day is almost invariably from atelectasis, the treatment of which requires active participation and cooperation from the patient. If atelectasis does not resolve, it leads to the development of pneumonia, which can be identified in chest x-ray films and confirmed with sputum cultures. At that time the process is no longer purely mechanical, but is also infectious, thus requiring antibiotics. Deep venous thrombosis **(choice A)** occurs about 5-7 days after surgery and is a “hidden” source of fever, i.e., nothing else seems to be wrong. This patient is clearly a candidate for thrombosis (he lies in bed doing nothing all day), but right now his problem is probably in the lung. The urine **(choice B)** is a good possibility when the fever starts on day 3, but the persistence of fever since day 1 points to the lung. Three days is too soon for a wound infection **(choice D)** to be the cause of the fever. Five to seven days is a more likely time frame. Looking for an abdominal abscess **(choice E)** is a little premature on the 3rd day after surgery. These typically take a week to 10 days to develop.
31
A middle-aged man with symptomatic carotid stenosis underwent a carotid endarterectomy on the right side. The area of significant stenosis extended from the carotid bifurcation up into the internal carotid, requiring a very high dissection and clamping of the vessel. The endarterectomy was done with an in situ shunt and closed with a Dacron patch. In the postoperative period, the patient has persistent difficulty swallowing solids and even more difficulty swallowing liquids. Any attempt to do so results in violent coughing and aspiration. His lips look symmetric and move normally, he speaks in a normal tone of voice without tiring, and he has no trouble breathing. When he is asked to stick his tongue out, he does so without deviation to either side. His symptoms are due to intraoperative damage of which of the following nerves? (A) Main trunk of the tenth (vagus) nerve (B) Mandibular branch of the seventh (facial) nerve (C) Sensory fibers of the ninth (glossopharyngeal) nerve (D) Superior laryngeal branch of the tenth (vagus) nerve (E) Trunk of the twelfth (hypoglossal) nerve
Respuesta: C The **correct answer is C**. Sensory fibers of the ninth (glossopharyngeal) nerve are in the vicinity of the digastric muscle, and can be damaged by retraction and dissection in the area. The lack of sensory input at the base of the tongue prevents the normal protective reflex that closes the glottis when swallowing liquids. Unilateral injury to the main trunk of the vagus **(choice A)** in the neck would produce symptoms from the recurrent fibers that innervate the larynx, producing a hoarse voice but no change in swallowing. Injury to the mandibular branch of the facial nerve **(choice B)** would produce drooping of the corner of the mouth and leaking of fluid at that level. Soup running out of the corner of the mouth is annoying, but swallowing is not affected. If the superior laryngeal branch of the vagus is damaged **(choice D)**, the voice tires easily. Swallowing is not affected. The twelfth nerve **(choice E)** is the most commonly damaged nerve during carotid endarterectomy because it crosses the internal and external carotids a short distance cephalad to the bifurcation. However, the outcome is deviation of the tongue to the affected side.
32
A 52-year-old man has been impotent ever since he had an abdominoperineal resection for cancer of the rectum. The tumor was staged as T3, N0, M0. He gets no nocturnal erections, and his impotence extends to all situations, regardless of sexual partner, and includes inability to masturbate. His erectile dysfunction is most likely due to which of the following? (A) Arterial vascular insufficiency (B) Erectile nerve damage (C) Psychogenic factors (D) Tumor invasion of the urethra (E) Venous incompetence
Respuesta: B The **correct answer is B**. A well-known risk of abdominoperineal resection is damage to the erectile nerves, as the rectum is widely dissected away from the pelvic walls. A 50% incidence of postoperative impotence is commonly quoted. “Nerve sparing” surgery can be done if the dissection is closer to the rectal wall, but the extent of the tumor may preclude it. A T3 (which this man had) is a large, bulky, primary tumor. Arterial insufficiency **(choice A)** will lead to impotence either of sudden onset after perineal trauma (motorcycle accident, for instance) or of very gradual development in chronic vascular disease. Psychogenic impotence **(choice C)** has a sudden onset but is partner- or situation-specific. Typically, nocturnal erections are preserved, as is the ability to masturbate. Tumor invasion **(choice D)**, if it is into the pelvic tissues, may indeed require a dissection that damages the nerves, but the problem would not involve the urethra. Venous incompetence **(choice E)** is a common source of organic impotence, which would have gradual onset and no connection with pelvic surgery.
33
A 62-year-old man who had a motorcycle accident has been in a coma for several weeks. He is on a respirator, has had pneumonia on and off, has been on pressors, and shows no signs of neurologic improvement. The family inquires about brain death and possible organ donation. An independent neurologic evaluation confirms that the patient is brain dead. What advice should be given to his family? (A) Anyone who has had pneumonia is excluded as a donor (B) He is not a suitable donor because of his age (C) Patients on respirators cannot donate organs (D) The harvesting team should evaluate him as a potential donor (E) The use of pressors precludes organ donation
Respusta: D The **correct answer is D**. Nowadays, when thousands of patients are awaiting transplants and many die before they get them, every potential donor should be evaluated by the experts. They may indeed reject some, but probably very few. For instance, corneas can be harvested even from people with cancer, and donors with chronic viral infections can be used for patients who have the same viral disease. All the other options were at one time or another used to exclude donors. These situations still make solid organs less attractive, but not to the extent that evaluation should be precluded.
34
A 22-year-old woman is taken to the emergency department after she injures her foot. She had been standing on a chair changing a light bulb, when she accidentally stepped off the chair backward. She heard a cracking sound when she fell and developed pain and swelling behind the ankle. Her symptoms worsened when she tried to descend the stairs in her house. Physical examination demonstrates marked swelling behind her ankle, and her pain is exacerbated by plantar flexion and dorsiflexion of the hallus. Which of the following is the most likely diagnoses? (A) Anterior Achilles tendon bursitis (B) Calcaneal spur syndrome (C) Epiphysitis of the calcaneus (D) Fracture of the posterolateral talar tubercle (E) Posterior tibial nerve neuralgia
Respuesta: The **correct answer is D**. This is a typical history for fracture of the posterior lateral talar tubercle; this fracture also occurs in basketball and tennis players who come down hard after a jump. This type of fracture is a fairly common foot injury that you should be able to clinically recognize. The diagnosis can be confirmed with lateral x-ray films of the ankle. Treatment is with immobilization in a cast for 4-6 weeks. Anterior Achilles tendon bursitis **(choice A)** would also cause pain behind the foot/ankle, but would develop more slowly. Calcaneal spur syndrome **(choice B)** causes heel pain and usually develops slowly. Epiphysitis of the calcaneus **(choice C)** is a painful cartilage break in the heel of young children in whom the two centers of ossification of the calcaneus have not yet fused. Posterior tibial nerve neuralgia **(choice E)** causes pain (sometimes burning) around the ankle and sometimes extending to the toes; the pain is worse on walking.
35
A 26-year-old, drug-addicted man develops congestive heart failure over a period of a few days. He is febrile, has a loud, diastolic murmur at the right second intercostal space, and has a blood pressure of 120/20 mm Hg. A physical examination performed a few weeks ago, when he attempted to enroll in a detoxification program, was completely normal. His blood pressure at that time was 120/80 mm Hg, and no murmurs were noted. In addition to long-term antibiotic therapy, which of the following is the most appropriate next step in management? (A) Closure of the ventricular septal defect with a pericardial patch (B) Elective aortic valve repair if he develops a systolic gradient of 50 mm Hg (C) Emergency aortic valve replacement (D) Emergency mitral valve repair (E) Emergency pulmonic valve replacement
Respuesta: C The **correct answer is C**. You probably had no trouble discerning that bacterial endocarditis, triggered by the use of nonsterile IV drugs, damaged a heart valve in this man. Although we usually think of the right-sided valves as the ones that are first in line to catch the bugs, any valve can become the seat of infection. The clinical presentation leaves no doubt that the aortic valve is the one that has been destroyed: the murmur is systolic, at the right base, and the very low diastolic pressure reveals the incompetence of the aortic valve. Furthermore, this was not a slow process allowing for compensation: he is in failure and close to death. He needs a new valve, pronto! Closure of the ventricular septal defect with a pericardial patch **(choice A)** would have been more appropriate for a massive myocardial infarction producing a septal defect. The clinical presentation would have been different, with a systolic murmur and normal diastolic pressure. Elective repair **(choice B)** is wrong for at least two reasons: it would not address an acute problem, and the criteria given were those for long-standing aortic stenosis (not insufficiency). The mitral valve **(choice D)** does not produce the symptoms described. The pulmonic valve **(choice E)** is not the culprit either. If it were destroyed, it would give few manifestations (it is almost a “disposable” valve).
36
An 18-year-old man was traveling at a high speed when his car slammed into a wall. He is brought into the emergency department by ambulance. His blood pressure is 60/40 mm Hg, pulse is 115/min and weak, respirations are 18/min, and central venous pressure is 2 cm H2O. He is responsive only to painful stimuli. Breath sounds are equal bilaterally, and cardiac auscultation reveals only tachycardia. The abdomen is soft, nondistended, and nontender with active bowel sounds. A chest x-ray film shows a widened mediastinum. Which of the following is the most likely diagnosis? (A) Cardiac contusion (B) Cardiac tamponade (C) Flail chest (D) Ruptured thoracic aorta (E) Tension pneumothorax
Respuesta: D The **correct answer is D**. This patient experienced a severe deceleration injury. He is hypotensive, tachycardic, and minimally responsive. He is in hemorrhagic shock. The chest x-ray reveals a widening mediastinum, suggesting a rupture of the thoracic aorta, which is a common catastrophic injury in deceleration accidents. This patient is in grave danger. After confirmation of the diagnosis by spiral CT scan, the treatment is immediate surgical repair of the injury with fluid and blood resuscitation. Cardiac contusion **(choice A)** is common in blunt-force injuries in which the steering wheel has crushed the chest. Arrhythmias, bundle branch block, or ECG abnormalities mimicking infection may occur. Pericardial effusion or rupture may develop. Cardiac tamponade **(choice B)** is associated with hypotension and tachycardia. However, pulsus paradoxus (systolic blood pressure drops >10 mm Hg on respiration) and distant heart sounds might be discovered on physical examination, and his central venous pressure would be high. Chest x-ray films would show an enlarged cardiac silhouette. The ECG would exhibit low limb-lead voltage and variable QRS amplitude (electrical alternans). However, pericardiocentesis is both the diagnostic and therapeutic procedure of choice. Flail chest **(choice C)** is diagnosed when a part of the chest wall bound by fractured ribs moves paradoxically during respiration. Ventilation is hampered. Tension pneumothorax **(choice E)** occurs when air can enter but not leave the pleural space. The mediastinum appears to be shifted to the contralateral side on chest x-ray films. A region without peripheral lung markings outlined by a sharp pleural margin is characteristic. Breath sounds are depressed or absent on the affected side.
37
A 72-year-old chronic smoker with severe chronic obstructive pulmonary disease (COPD) is found to have a central hilar mass on chest x-ray. Bronchoscopy and biopsies establish a diagnosis of squamous cell carcinoma of the lung. Pulmonary function studies show that he has an FEV1 of 1100 mL, and a ventilation-perfusion scan indicates that 60% of his pulmonary function comes from the affected lung. Which of the following is the most appropriate next step in management? (A) CT scan of the upper abdomen to rule out liver metastasis (B) Mediastinoscopy to biopsy carinal nodes (C) Radiation and chemotherapy (D) Palliative pneumonectomy (E) Pneumonectomy with hope of cure
Respuesta: C The **correct answer is C**. This man is not a surgical candidate, thus ruling out pneumonectomy **(choices D and E)**. With a central lesion, he would require a pneumonectomy rather than a lobectomy. After resectional pulmonary surgery is done, however, a patient must be left with at least 800 mL in the FEV1 to live a semi-decent life. Anything less than that would make him a pulmonary cripple, or outright kill him. Because of his COPD, this patient is already severely limited, with a total FEV1 of 1100 mL. Were the bad lung to be removed, he would be left with only 40% of 1100 mL: 440 mL. The only option left is radiation and chemotherapy. CT scan of the upper abdomen to rule out liver metastasis **(choice A)** and mediastinoscopy to biopsy carinal nodes **(choice B)** are necessary steps to establish curability. There is no point in doing a pneumonectomy if there are liver or carinal node metastases. But if a pneumonectomy cannot be done for reasons of poor function, there is no point in finding out whether the other limiting factors are present.
38
A 14-year-old boy presents in the emergency department with very severe pain of sudden onset in his right testicle. There is no history of either trauma or recent mumps. He is afebrile, and a urinalysis shows no pyuria. The testis is swollen, exquisitely painful, high in the scrotum, and riding in a horizontal position. The cord above the testis is not tender. Which of the following is the most appropriate next step in management? (A) Ice packs, analgesics, and careful observation (B) Sonogram of the testicle (C) IV antibiotics (D) Testicular biopsy (E) Emergency surgery
Respuesta: E The **correct answer is E**. The child has testicular torsion, one of the very few true urologic emergencies. He needs immediate de- torsion if the testis is to be saved. No time should be wasted doing further studies. Symptomatic care **(choice A)** is fine for testicular trauma with scrotal hematomas. In this case, it would amount to malpractice. A sonogram **(choice B)** is always done when the clinical diagnosis is epididymitis, and we want to be sure that torsion is not being overlooked. But when the clinical diagnosis screams “torsion,” as in this vignette, time wasted confirming the diagnosis with the sonogram could lead to loss of the testicle. Antibiotics **(choice C)** are effective therapy for acute epididymitis, the condition with which testicular torsion may be confused. But the patient with epididymitis is usually somewhat older (sexually active) and has fever, pyuria, a very tender cord, and a normally positioned testicle. Testicular biopsy **(choice D)** is done when we think that the diagnosis is cancer, but the scenario would be a painless mass in a young male.
39
An 80-year-old man comes to the physician because of a slowly growing ulcerated mass on the glans penis. A biopsy is positive for squamous cell carcinoma. Which of the following conditions is usually present in association with this tumor? (A) Balanitis xerotica obliterans (B) Condyloma acuminatum due to human papillomavirus (HPV) type 6 (C) Lack of circumcision (D) Peyronie disease (E) Syphilis
Respuesta: C The **correct answer is C**. Squamous cell carcinoma of the penis is admittedly a rare form of cancer in the U.S. (about 1% of all cancers in males). It is virtually unknown in nations that practice early (i.e., in infancy) circumcision. It presents as a fungating or ulcerated mass on the glans penis or in the sulcus between the prepuce and the glans. Carcinogenic agents forming in the smegma are suspected to play a pathogenic role, which may be enhanced by lack of circumcision. Other risk factors include lesions (condyloma acuminatum) due to human papillomavirus (HPV) types 16 and 18. Genomic material from these HPV types has been demonstrated in numerous cases. No association has been observed between penile cancer or its precursors and other types of HPV, such as type 6 **(choice B)** or 11. Psoriasis is associated with a slightly increased risk. Balanitis xerotica obliterans **(choice A)** results in fibrosis and thickening of the prepuce. This condition does not seem to confer a significantly increased risk of developing penile carcinoma. Peyronie disease **(choice D)**, which is a form of fibromatosis affecting the penis, results in induration, nodularity, and deformities. It causes severe functional deficits but is not associated with increased risk of cancer. This condition is akin to other forms of fibromatosis, such as Dupuytren contracture. The primary stage of syphilis **(choice E)** manifests with a painless, sharply demarcated ulcer (chancre) often located in the glans. There is no association between syphilis and penile cancer.
40
A 45-year-old woman with breast cancer undergoes a modified radical mastectomy with lymph node dissection. Six weeks later, she returns complaining of decreased mobility of her shoulder. On physical examination, the scapula protrudes from the body when pressing her outstretched arm on the wall. Which of the following nerves was most likely injured during the operation? (A) Intercostal (B) Lateral pectoral (C) Long thoracic (D) Medial pectoral (E) Thoracodorsal
Respuesta: C The **correct answer is C**. This patient has scapular “winging,” a protrusion of the scapula when the ipsilateral outstretched arm is pressed against a wall. Scapular winging results from paralysis of the serratus anterior muscle, which functions to carry the scapula forward and assist the deltoid in raising the arm. The serratus is innervated by the long thoracic nerve, which is derived from the fifth, sixth, and seventh cervical nerve roots. This woman had an axillary lymph node dissection that may have damaged the inferior part of the brachial plexus, leading to long thoracic nerve damage, paralysis of the serratus, and scapular winging. The intercostal nerves **(choice A)** arise from the thoracic nerve roots and innervate the intercostal muscles of the chest wall. The lateral pectoral **(choice B)** and medial pectoral **(choice D)** nerves innervate the pectoralis major and minor muscles. The thoracodorsal nerve **(choice E)** innervates the latissimus dorsi muscle.
41
A young man is brought to the emergency department following a head-on collision at 30 miles per hour. He is awake and alert. Other than a forehead laceration, physical examination is normal and laboratory values are within normal limits. Chest x-ray films are unremarkable. Which of the following is the most appropriate next step in diagnosis? (A) Echocardiogram (B) Lateral cervical spine x-ray (C) CT scan of the abdomen (D) CT scan of the head (E) Peritoneal lavage
Respuesta: B The **correct answer is B**. In patients sustaining trauma, there is a chance of bony cervical spine injury. Iatrogenic injury is to be avoided by immobilization of the cervical spine until bony injury is ruled out by lateral cervical spine x-ray. An echocardiogram **(choice A)** would not be appropriate with a normal physical examination and no suspicious cardiac findings. CT scan of the abdomen **(choice C)** or peritoneal lavage **(choice E)** are useful for evaluating the degree of abdominal injury when abdominal examination is inadequate, especially in an unconscious trauma patient. CT examination of the head **(choice D)** is required for patients with altered mental status or for those in whom an intracranial bleed is suspected.
42
A 51-year-old man is undergoing abdominal surgery and becomes hypotensive while under general anesthesia. The patient had been doing well during most of the procedure but now has a blood pressure of 80/40 mm Hg. His past medical history is significant for coronary artery disease and diabetes mellitus. A pulmonary artery catheter placed prior to the gives the following data: - Central venous pressure 10 mm Hg - Pulmonary artery pressure 60/30 mm Hg - Pulmonary capillary occlusion pressure 24 mm Hg - Cardiac output 2.3 L/min Which of the following is the most likely diagnosis? (A) Acute left heart failure (B) Acute mitral regurgitation (C) Acute right heart failure (D) Hypoxic pulmonary vasoconstriction (E) Sepsis syndrome
Respuesta: A The **correct answer is A**. Pulmonary artery (Swan-Ganz) catheters are ubiquitous in critical care settings; a basic ability to interpret data from them is vital to the practice of inpatient hospital medicine. This patient has a low cardiac output and a high filling pressure (>18 mm Hg) and is hypotensive. Therefore, this patient’s shock syndrome is cardiogenic. Cardiogenic shock is caused by a number of underlying problems, but the end result is left ventricular failure. This also accounts for the secondarily high rightsided pressures and filling pressures (left heart failure causes right heart failure). Acute mitral regurgitation **(MR; choice B)** is a possibility in this case. At first glance, acute MR could account for all of the patient’s findings, both on physical examination and pulmonary artery catheter. However, unless the acute MR occurred in the setting of acute ischemia, there is no way to account for the severely depressed cardiac output (LV function). Therefore, isolated acute MR from a papillary muscle rupture or chordae rupture could not alone account for all of this patient’s findings. Acute right heart failure **(choice C)** is incorrect because it fails to explain the elevated left-sided filling pressures. Hypoxic pulmonary vasoconstriction **(choice D)** would acutely produce elevated pulmonary artery pressures and possibly right heart failure over the long term, but not in an acute manner. Sepsis syndrome **(choice E)** is defined as hyperdynamic cardiac output (supraphysiologic) with systemic hypotension. This patient has a depressed output, not compatible with sepsis.
43
An 18-year-old gang member is stabbed in the back, just to the right of the midline. Physical examination shows paralysis and loss of proprioception distal to the injury on the right side, and loss of pain perception distal to the injury on the left side. Which of the following is the most likely diagnosis? (A) Anterior cord syndrome (B) Central cord syndrome (C) Complete transection of the spinal cord (D) Hemisection of the spinal cord (E) Posterior cord syndrome
Respuesta: D The **correct answer is D**. Even if you do not remember all the spinal cord tracts and pathways, you should recognize that deficits in one set of functions below the injury on one side, combined with deficits in a different group of functions on the other side, spells out hemisection. Furthermore, such clean-cut division of one side of the spinal cord can happen only with a knife or a bullet, which is the setting in this vignette. Anterior cord syndrome **(choice A)** is characterized by sparing of the posterior columns, with preservation of vibration and position sense. The etiology is typically vascular. Central cord syndrome **(choice B)** is seen in whiplash injuries, in which it produces severe deficits in the upper extremities, with relative sparing of the lower extremities. Sensory loss consists of loss of pain and temperature sense in a “cape” distribution over the shoulders, lower neck, and upper trunk with posterior column function (light touch, vibration, conscious pro-prioception) relatively preserved. Complete transection **(choice C)** would result in spastic paralysis below the level of the injury with flaccid paralysis at the level of the injury. Ascending sensory fibers would also be interrupted, producing a total sensory deficit below the level of the lesion. Posterior cord syndrome **(choice E)** would result in selective loss of conscious proprioception, vibratory sense, stereognosis, graphesthesia, and two point discrimination. It is fairly rare.
44
A 31-year-old man is brought by helicopter to the trauma center after a motor vehicle accident in which he sustained massive lower extremity crush injury. The patient is alert and awake but in tremendous pain. His blood pressure is 140/80 mm Hg, and his pulse is 110/min. There is copious ongoing blood loss from the sites of injury. Urgent laboratory data will most likely show which of the following electrolyte abnormalities? (A) Hyperkalemia (B) Hypernatremia (C) Hypocalcemia (D) Hypoglycemia (E) Hypophosphatemia
Respuesta: A The **correct answer is A**. Crush injury results in massive tissue damage. There are obvious organ-specific consequences of such injury, but it is the musculoskeletal system that is the focus of this question. Massive necrosis and lysis of muscle releases intracellular myoplasm components that are toxic in high concentrations. Examples of such constituents include potassium, creatine kinase, and protein. It can be generally assumed that any patient with significant crush injuries will have autoinfusion of hundreds of milliequivalents of potassium into the blood. There is no significant derangement in sodium levels, such as hypernatremia **(choice B)**, after a crush injury. Cells in general, and myocytes in particular, are storehouses of calcium. With significant myonecrosis, serum calcium can rise to dangerous levels, producing hypercalcemia, rather than hypocalcemia **(choice C)**. Hypoglycemia **(choice D)** would not be seen in this patient. Because of the stress of his injuries and the elevated levels of cortisol and epinephrine in his blood, massive amounts of glucose would be mobilized. Thus, if any derangement in glucose homeostasis were present, it would likely be hyperglycemia. Hyperphosphatemia, rather than hypophosphatemia **(choice E)**, is observed with a crush injury. Phosphate is an intracellular buffer that is also released from cells after necrosis and lysis.
45
A 32-year-old woman has an episode of upper gastrointestinal bleeding after a night of heavy alcoholic intake followed by ingestion of multiple aspirin tablets for the hangover. There was no prior vomiting until the time when she felt nauseated, went to the bathroom, and filled the wash basin with vomiting of bright red bloody fluid.” When she arrives in the emergency department, an upper gastrointestinal endoscopy is promptly performed, which confirms a diagnosis of acute erosive gastritis. She has no duodenal ulcer and no esophageal varices. Gastric lavage with ice-cold saline is performed and the bleeding stops. Laser photocoagulation or electrocautery are not used, neither is pitressin infused. She remains hemodynamically stable throughout the procedure, and she has a normal hemoglobin. She is sent home 2 hours later. Four hours after discharge, she returns complaining of severe, constant chest pain. She is in acute distress, has a temperature of 39.0 C (102.2 F), is having chills, and looks quite ill. Physical examination is remarkable for the presence of crepitation to palpation in the upper chest and lower neck, and chest x-rays confirm the presence of air in the mediastinum and the subcutaneous tissues. Which of the following is the most likely diagnosis? (A) Boerhaave syndrome (B) dissecting thoracic aortic aneurysm (C) gastric perforation (D) iatrogenic esophageal perforation (E) myocardial infarction
Respuesta: D The **correct answer is D**. Acute mediastinitis with air in the tissues that occurs within a few hours of an upper gastrointestinal endoscopy is virtually diagnostic of instrumental, iatrogenic esophageal perforation. Boerhaave syndrome **(choice A)** is also a form of esophageal perforation, but it is caused by protracted, forceful vomiting, which this patient did not have. A dissecting aneurysm of the thoracic aorta **(choice B)** can mimic a myocardial infarction, but it would not fill the mediastinum with air. In upper gastrointestinal endoscopy, the esophagus, which has narrow lumen and a flimsy wall, can be perforated easily, but the stomach (choice C), which has ample lumen and a thick, strong wall, almost never is. Furthermore, other than looking, nothing was done that could damage the stomach (laser or electrocoagulation). Finally, a hole in the stomach gives an acute abdomen with free air under the diaphragm, rather than mediastinitis with air in the mediastinum. A real myocardial infarction (choice E) is rare in a 32-year-old, and like the previous option, it produces pain but not mediastinitis with air.
46
A 45-year-old man comes to the emergency department because of severe right flank pain that began abruptly 3 hours ago. The pain comes in waves and radiates down to the ipsilateral testis. The patient is nauseated and extremely restless. His temperature is 37.0 C (98.6 F). Dipstick examination of urine is positive for hematuria. Urinary pH is 5.8. Which of the following is the most appropriate next step in diagnosis? (A) Intravenous pyelography (IVP) (B) Plain abdominal x-ray film (C) Renal ultrasound examination (D) Serum calcium, phosphorus, electrolytes, and uric acid (E) Urine cultures
Respuesta: B The **correct answer is B**. A plain abdominal x-ray film is most likely to detect a stone in this patient, who manifests the typical symptomatology of renal colic, most commonly due to a urinary stone impacted in the ureter. Usually, gross or microscopic hematuria is present. Absence of fever is an important negative sign excluding coexistence of urinary tract infection. Most urinary stones consist of calcium phosphate or oxalate and thus contain enough calcium to be visible on plain xray films. However, some urinary calculi are radiolucent (especially uric and cystine stones), whereas others are so small as to be undetectable on plain x-ray films. Intravenous pyelography **(IVP; choice A)** is rarely necessary in patients with the typical presentation of renal colic. However, IVP becomes necessary when the diagnosis is uncertain. Frequently, this investigation will demonstrate dilatation of the ureter proximal to the site of stone blockage. Renal ultrasound examination **(choice C)** is useful when the stone is suspected to be located at the ureterovesical junction. The bladder should be full to allow ultrasonography. Serum calcium, phosphorus, electrolytes, and uric acid **(choice D)** should be evaluated in patients experiencing a first urinary tract stone, but serum chemistry studies are not necessary as initial diagnostic investigations. Urine cultures **(choice E)** are not necessary for uncomplicated urinary stone disease without clinical evidence of urinary tract infection. However, abnormal urinary pH should suggest participation of infectious agents in stone formation. Proteus infections result in alkaline pH because of urease production. On the other hand, a pH lower than 5.0 suggests urate or cystine stones.
47
A 71-year-old man is involved in a minor automobile accident on the road between Guadalajara and Lake Chapala in Mexico. The man is an American citizen who at the age of 65 years retired to a lakeside home in that area. Although he is asymptomatic, he decides to return to the United States to be “thoroughly checked.” He is admitted to a veteran’s hospital in south Texas, where he undergoes a CT scan of his abdomen. There are no signs of traumatic injuries, but the scan reveals the presence of four simple, thin walled cystic structures, approximately 1 cm in diameter, scattered throughout both lobes of his liver. They have no septations. There are no cysts in the kidneys or pancreas. The man is completely asymptomatic and afebrile. Liver function tests are normal, as is his white blood count and differential. Which of the following is the most likely diagnosis? (A) Amebic abscesses (B) Cystadenocarcinoma of the liver (C) Hydatid cysts (D) Polycystic liver disease (E) Simple liver cysts
Respuesta: E The **correct answer is E**. The description is perfect for simple liver cysts, which occur in roughly 10% of the population at large. Nothing needs to be done about this. The “Mexico connection” is the tip-off to suspect amebic abscess **(choice A)** in a patient who lives there and who presents with fever, leukocytosis, a tender liver, and elevated alkaline phosphatase. That is not the clinical picture this patient has, however. Cystadenocarcinomas **(choice B)** have thick walls with multiple septations. Calcifications in the cyst wall also may be present, and usually the affected patient is a middle-aged woman. Hydatid cysts **(choice C)** can occur but are not particularly prevalent in Mexico, a little geographic detail unknown to most American physicians. High plains in the Andes or a Mediterranean country would be a better location. In any event, however, the hydatid cyst is characteristic, with a large mother cyst containing daughter cysts around the periphery. The cysts of polycystic liver disease **(choice D)** occur in much larger numbers, tend to enlarge over the years, and often are accompanied by renal cysts.
48
A 27-year-old woman who moved to the U.S. from Southeast Asia at the age of 10 presents with gross hematuria. She reports chronic low-grade fever and weight loss for over 1 year. Urinalysis also shows pyuria, but urinary cultures are negative for bacteria. Intravenous pyelography (IVP) reveals diminished contrast excretion in the right kidney and cavitary lesions in the right kidney. (A) Acute postinfectious glomerulonephritis (B) Autosomal dominant polycystic kidney disease (C) Bacterial cystitis (D) Berger disease (E) Bladder transitional cell carcinoma (F) Hemorrhagic cystitis (G) Prostatic carcinoma (H) Testicular cancer (I) Transitional cell carcinoma of the pelvis (J) Tuberculosis (K) Ureteral stone (L) Urethral carcinoma (M) von Hippel-Lindau syndrome
Respuesta: J The **correct answer is J**. Secondary tuberculosis commonly affects the kidneys by hematogenous dissemination. Cavitary lesions may be demonstrated by intravenous pyelography (IVP). The patient frequently gives a history of weight loss and low-grade fever. The infection spreads with the urinary flow down into the bladder, frequently involving the epididymis and prostate in men. Sometimes, the first clinical manifestation of urinary tract tuberculosis is hematuria. Urine cultures are often negative, but the urine contains large numbers of leukocytes (so-called sterile pyuria).
49
A 35-year-old woman with corticosteroid-resistant systemic lupus erythematosus is being treated with cyclophosphamide. She presents with increased frequency of urination and gross hematuria. Urinalysis is negative for pyuria or bacteriuria. (A) Acute postinfectious glomerulonephritis (B) Autosomal dominant polycystic kidney disease (C) Bacterial cystitis (D) Berger disease (E) Bladder transitional cell carcinoma (F) Hemorrhagic cystitis (G) Prostatic carcinoma (H) Testicular cancer (I) Transitional cell carcinoma of the pelvis (J) Tuberculosis (K) Ureteral stone (L) Urethral carcinoma (M) von Hippel-Lindau syndrome
Respuesta: F The **correct answer is F**. Hemorrhagic cystitis is characterized by marked hematuria, resulting from extensive erosions of the urinary bladder mucosa. Cyclophosphamide is one of the most common etiologic agents of this form of cystitis. Another cause of hemorrhagic cystitis is radiation.
50
A 35-year-old man with prior history of cerebellar hemangioblastoma and multiple cysts in the pancreas is found to have microhematuria on a routine check-up. One of his siblings developed hemangioblastomas and renal cell carcinoma in his youth, and similar cases are traceable on his father’s side of the family. Renal ultrasonography reveals a 4-cm partially cystic mass in the right kidney, and a similar smaller lesion in the left kidney. (A) Acute postinfectious glomerulonephritis (B) Autosomal dominant polycystic kidney disease (C) Bacterial cystitis (D) Berger disease (E) Bladder transitional cell carcinoma (F) Hemorrhagic cystitis (G) Prostatic carcinoma (H) Testicular cancer (I) Transitional cell carcinoma of the pelvis (J) Tuberculosis (K) Ureteral stone (L) Urethral carcinoma (M) von Hippel-Lindau syndrome
Respuesta: M The **correct answer is M**. Von Hippel-Lindau syndrome is a hereditary autosomal dominant disease characterized by hemangioblastomas of the CNS (most frequently in the cerebellum and retina), cysts of the pancreas and kidneys, and renal cell carcinoma. The patient’s family history is perfectly compatible with this diagnosis. Thus, the presence of hematuria, along with coexistent renal lesions detected on ultrasonography, indicates that renal cell carcinoma has also developed in this patient. Renal cell carcinoma associated with von Hippel-Lindau syndrome is often multicentric and cystic. Recall that the gene involved in this disease (VHL gene) has been found mutated in most cases of the more common sporadic renal cell carcinoma. Acute postinfectious glomerulonephritis **(choice A)** manifests with nephritic syndrome, characterized by hematuria, hypertension, proteinuria less than 3 g/day, and mild edema (periorbital and pedal). It usually occurs in children 1-3 weeks following group A Streptococcus pharyngitis or impetigo. Autosomal dominant polycystic kidney disease **(choice B)** manifests in middle-aged or elderly individuals with hematuria and hypertension. Usually, palpable enlarged kidneys are found on physical examination. Bacterial cystitis **(choice C)** is commonly due to Escherichia coli. Dysuria, frequency of urination, and suprapubic discomfort are the principal symptoms. Hematuria, gross or microscopic, is frequently seen on dipstick urinalysis. Berger disease **(choice D)** is the most frequent cause of glomerulonephritis, manifesting with hematuria and/or proteinuria. It is often discovered incidentally. IgA deposition in the mesangium and high serum IgA levels are the defining features. Bladder transitional cell carcinoma **(choice E)** is associated with hematuria, but usually not with other symptoms. Intravenous pyelography (IVP) is not sensitive enough to detect this type of cancer. Urinary cytology and cystoscopy are necessary for diagnosis. Prostatic carcinoma **(choice G)** rarely manifests with hematuria as its first presenting sign. The most common mode of presentation nowadays is by elevated prostatic-specific antigen (PSA) and/or abnormal findings on digital rectal examination. Testicular cancer **(choice H)** is not associated with hematuria. Its most common first sign is painless testicular enlargement, associated with a sensation of heaviness. Frequently, patients come to the physician many months after the initial onset of these signs. Transitional cell carcinoma of the pelvis **(choice I)** is relatively rare compared with renal cell carcinoma and urinary bladder transitional carcinoma. Its most common presentation is hematuria, but IVP demonstrates these tumors as filling defects in the pelvicaliceal system. A ureteral stone **(choice K)** most commonly presents with renal colic, a paroxysmal form of flank pain associated with extreme restlessness that results from ureteral obstruction. Hematuria is present during colicky episodes and sometimes in asymptomatic periods. IVP would show dilatation of the ureter proximal to the impacted stone. Urethral carcinoma **(choice L)** usually comes to medical attention because of bloody urethral discharge in an elderly patient. If hematuria is present, it is detected in the first few milliliters of urine (initial hematuria) when urine is collected in separate aliquots.